LSAT Prep Test 53 (December 2007) - S1 - Question 08

NathanialNathanial Alum Member
Hey guys,

In this weakening situation I heard JY say that answer choice D is a rare one that attacks the premise (5:04). Just wanted to know when is it ok or acceptable in a weakening situation to attack the premise.

Thanks

Comments

  • nye8870nye8870 Alum
    1749 karma
    He says the premise's assertion still remains but because the studies did not have enough participants the studies' results are unreliable. I would still stick to the sop of not attacking premise directly.
Sign In or Register to comment.